Đến nội dung

Ego nội dung

Có 288 mục bởi Ego (Tìm giới hạn từ 29-04-2020)



Sắp theo                Sắp xếp  

#616625 Xét một số nguyên dương n > 3 thì nó có thể được tách thành tối đa bao nh...

Đã gửi bởi Ego on 23-02-2016 - 22:08 trong Số học

Ta sẽ đặt $f(n)$ là hàm số thỏa mãn với $f: \mathbb{N*} \to \mathbb{N}$ sao cho $f(n)$ là số hợp số tối đa mà $n$ có thể phân tích được.
Dễ thấy $f(1) = f(2) = f(3) = 0$
TH1. $n = 4t$, khi đó để ý là $4$ là hợp số bé nhất. Khi đó dễ thấy $f(n) = t$.
TH2. $n = 4t + 1$. Ta sẽ chứng minh quy nạp là $f(4t + 1) = t - 1$. Kiểm tra được $f(4.1 + 1) = 0, f(4.2 + 1) = 1$. Giả sử khẳng định đúng với $t - 1$, nghĩa là $f(4t - 3) = t - 2$. Ta sẽ chứng minh là $f(4t + 1) = t - 1$. Thật vậy, với $4t + 1 = 4 + (4t - 3)$ do đó $f(4t + 1) \ge 1 + f(4t - 3) = t - 1$. Mặt khác, để ý là hợp số bé nhất là $4$, do đó số cách phân tích ra hợp số luôn bé hơn $\left \lfloor \frac{4t + 1}{4} \right \rfloor$, hay nói cách khác $f(4t + 1) < t$. Do $f(4t + 1)$ nhận giá trị nguyên nên $f(4t + 1) \le t - 1$. Từ đó kết hợp ta có $f(4t + 1) = t - 1$
TH3. $n = 4t + 2$. Ta cũng sẽ chứng minh quy nạp là $f(4t + 2) = t - 1$. Kiểm tra được $f(6) = 1, f(10) = 2$. Giả sử mệnh đề đúng với $t - 1$. Ta sẽ chứng minh $f(4t + 2) = t - 1$. Có $4t + 2 = 4(t - 1) + 6 \implies f(4t + 2) \ge t - 1$. Mặt khác, lí luận tương tự trên, có điều phải chứng minh.
TH4. $n = 4t + 3$.Ta cũng sẽ chứng minh quy nạp $f(4t + 3) = t - 1 \; \forall t \ge 3$ và $f(7) = f(11) = 0$. Kiểm tra được $f(7) = f(11) = 0$. Ta kiểm tra được $f(15) = 2, f(19) = 3$. Giả sử khẳng định đúng với $t - 1 \ge 3$, nghĩa là $f(4t - 1) = t - 2$. Khi đó $4t + 3 = 4 + (4t - 1)$ hay $f(4t + 3) \ge t - 1$. Tương tự trên, ta cũng có điều phải chứng minh.
Để cho chắc chắn, ta sẽ đưa ra các ví dụ $4t = 4 + \cdots + 4$, $4t + 1 = 9 + 4.(t - 2)$, $4t + 2 = 4.(t - 2) + 6$, $4t + 3 = 6 + 9 + 4(t - 3)$
 




#626782 Xác định tập hợp liên quan đến số chính phương mod $p$

Đã gửi bởi Ego on 12-04-2016 - 16:54 trong Số học

Cho $p$ là một số nguyên tố lẻ và hai tập con phân biệt không rỗng $A,B$ của tập $S= \{ 1,2, \cdots , p-1 \}$ thoả mãn:

 

1) $A \cup B=S$.

2) Nếu $a,b$ cùng thuộc tập $A$ hoặc cùng thuộc tập $B$ thì $ab \in A$.

3) Nếu $a,b$ thuộc hai tập khác nhau $A,B$ thì $ab \in B$.

 

Tìm tất cả các tập $A,B$ có thể.

Mình có hai câu hỏi

i) $ab$ có nghĩa là $ab \pmod{p}$ đúng không nhỉ?
ii) $a, b$ có nhất thiết phân biệt không  :luoi:




#619753 Xác định $P(n + 2)$

Đã gửi bởi Ego on 11-03-2016 - 20:27 trong Đa thức

Cho đa thức $P(x)$ bậc $n$ thỏa mãn $P(n) = \frac{1}{n} \; \forall n = 1, 2, \cdots n + 1$. Xác định $P(n + 2)$.




#622984 về các số $7^{2n}+5.7^n+1$

Đã gửi bởi Ego on 27-03-2016 - 17:51 trong Số học

Không ai giải hết  :( 
Lời giải của mình. Gọi $p$ là một ước nguyên tố của $7^{4k + 2} + 5.7^{2k + 1} + 1$. Nhận xét $p\neq 7$ và $p\neq 3$ và $p$ lẻ:
i) Ta có $p\mid (7^{2k + 1} + 1)^{2} + 21.7^{2k} \implies p\mid [7^{-k}(7^{2k + 1} + 1)]^{2} + 21 \implies \left(\frac{-21}{p}\right) = 1$
ii) Lại có $p\mid 4(7^{2n} + 5.7^{n} + 1) = (2.7^{n} + 5)^{2} - 21 \implies \left(\frac{21}{p}\right) = 1$
Kết hợp lại, ta có $1 = \left(\frac{-21}{p}\right) = \left(\frac{21}{p}\right).\left(\frac{-1}{p}\right) = \left(\frac{-1}{p}\right)$.
Từ đó ta suy ra $p \equiv 1\pmod{4}$
Chứng minh hoàn tất. $\bigstar$




#637710 VMF's Marathon Đa thức Olympic

Đã gửi bởi Ego on 02-06-2016 - 22:59 trong Đa thức

Bài toán 1 hiện vẫn chưa có lời giải nên mình được phép đề xuất bài toán tiếp theo:
Bài toán 2. Cho $P(x) \in \mathbb{Z}[x]$ thỏa mãn $P(x)$ là số chính phương với mọi $x$ nguyên thì $P(x) = Q(x)^{2}$ với $Q(x) \in \mathbb{Z}[x]$.
Mở rộng: Liệu bạn có thể giải quyết cho trường hợp $P(x)$ là lũy thừa bậc $n$ của một số nguyên thì $P(x) = Q(x)^{n}$ với $Q(x)\in\mathbb{Z}[x]$




#636860 VMF's Marathon Đa thức Olympic

Đã gửi bởi Ego on 30-05-2016 - 17:53 trong Đa thức

Bài toán hiện nay:

Bài toán 1 : Tìm tất cả các đa thức số hệ số nguyên thỏa mãn $a+b$ là số chính phương thì $P(a)+P(b)$ cũng là số chính phương, trong đó $a, b$ là các số tự nhiên (bangbang1412)
Bài toán 2. Cho $P(x) \in \mathbb{Z}[x]$ thỏa mãn $P(x)$ là số chính phương với mọi $x$ nguyên thì $P(x) = Q(x)^{2}$ với $Q(x) \in \mathbb{Z}[x]$.
Mở rộng: Liệu bạn có thể giải quyết cho trường hợp $P(x)$ là lũy thừa bậc $n$ của một số nguyên thì $P(x) = Q(x)^{n}$ với $Q(x)\in\mathbb{Z}[x]$ (Ego)




#636854 VMF's Marathon Đa thức Olympic

Đã gửi bởi Ego on 30-05-2016 - 17:44 trong Đa thức

Sau khi thông qua ý kiến các bạn, mình xin được nối tiếp các cuộc Marathon trước với Marathon Đa thức lần này. Mục tiêu là nhằm trau dồi, rèn luyện cho các bạn muốn tham gia các kỳ thi Olympic phổ thông.

Các chủ đề tiêu biểu mà các bạn có thể thảo luận:

  • Tính chia hết của đa thức
  • Quy tắc dấu Decartes
  • Các bài toán giải tích liên quan đến đa thức
  • Đa thức số học
  • Đa thức nguyên và các đa thức nhận giá trị nguyên
  • Tính bất khả quy/khả quy của đa thức
  • Phương trình hàm đa thức
  • Các bài toán xác định đa thức dựa trên các đặc trưng số học/các công thức nội suy
  • Đa thức lượng giác: Đa thức Chebyshev loại I, loại II
  • Các bài toán về đa thức nhiều biến
  • ...

Nội dung của cuộc thi này khá đơn giản, khi bạn giải đúng được bài toán hiện có thì bạn có thể đăng lên tại đây và mình sẽ cộng thêm cho các bạn một điểm, và các bạn có quyền được đề xuất bài toán mới. Như vậy ai giải thì người đó sẽ có quyền đề xuất, trừ khi bạn không biết đề xuất bài nào thì bạn có thể nhờ hỗ trợ.

 

Và một số quy định yêu cầu các bạn tuân thủ:

  1. Chỉ cho phép các bài toán trong phạm vi đa thức
  2. Ghi nguồn bài toán rõ ràng
  3. Không được phép giải bài toán của chính mình đề xuất, không được phép đề xuất các bài toán trong các cuộc thi chưa kết thúc (ví dụ như tạp chí toán học & tuổi trẻ,...)
  4. Không được spam, lời giải rõ ràng, cụ thể.
  5. Khi bạn giải bài toán thứ $n$ thì bạn đề xuất luôn bài toán thứ $n + 1$ (đánh đúng số thứ tự). Sau đây là mẫu:
    Lời giải bài $n$. ABCXYZ
    Bài toán $n + 1$. (Nguồn) Cho ba số $a, b, c$. Chứng minh rằng $3\mid abc$.
  6. Lưu ý không đăng các bài toán mở, các giả thuyết, ...
  7. Nếu một bài toán trong vòng $7$ ngày chưa ai giải được thì sẽ được đánh dấu lại và mình sẽ đăng bài toán tiếp theo. Bất cứ lúc nào bạn muốn đề xuất lời giải cho bài chưa được giải cũng được và sẽ được cộng hai điểm nếu như lời giải đúng. Ngoài ra nếu các bạn nghĩ mình có lời giải hay hơn của bạn trước tiên giải bài nào đó thì xin cứ đăng (sẽ chỉ cộng điểm cho bạn làm đúng và nhanh nhất), như vậy sẽ học hỏi lẫn nhau được nhiều hơn.
    Ngoài ra, trước khi hết hạn $4$ ngày của một bài toán chưa được giải thì mong các bạn không đề xuất bài toán mới.
  8. Yêu cầu các bài toán có độ khó nhất định, phải suy nghĩ mới làm được.
  9. Yêu cầu tuân thủ các quy định. Bài viết nào có tính chất spam sẽ bị xóa đi hoặc lời giải đúng nhưng không rõ ràng, lan man sẽ chỉ nhận được $0,5$ điểm.

Mình khuyến khích mọi người tự đưa lời giải của chính mình thay vì lời giải của người khác hoặc dẫn link lời giải.

Hi vọng các bạn tham gia và đón nhận  :D. Nếu các bài toán hay và lời giải đẹp thì ta sẽ tổng hợp thành một tài liệu nhỏ để tham khảo trong quá trình học Olympic, sẽ khá tốt.

Lưu ý: Các bạn khi đăng lời giải hãy để mọi người kiểm tra hộ bạn rồi hẳn đề xuất bài toán mới (kinh nghiệm của mình)
 




#638703 VMF's Marathon Đa thức Olympic

Đã gửi bởi Ego on 07-06-2016 - 12:54 trong Đa thức

Bài toán 4. Tìm $P(x)\in \mathbb{Z}_{[x]}$ sao cho $P(x)$ nguyên tố với $\forall x\in \mathbb{N*}$.

Lời giải bài 4. Ta sẽ chứng minh $P(x) = p \quad\forall x \in \mathbb{R}$ là nghiệm duy nhất. Thật vậy, giả sử tồn tại $P$ khác hằng sao cho $\deg P \ge 1$. Ta có thể giả sử $P* > 0$ nhờ vào việc xét $-P(x)$ (cái này như nhau).
Vì vậy, với $x$ đủ lớn thì $P(x) > 1$. Để ý là $P(x + P(x)) - P(x) \vdots P(x) \implies P(x + P(x)) \vdots P(x)$. Điều này mâu thuẫn với giả thiết số nguyên tố.
Bài toán 5. (Irish MO 27th) Cho $n$ là số nguyên dương và $a_{1}, a_{2}, \cdots a_{n}$ là các số thực dương. Đặt $g(x) = (x + a_{1})(x + a_{2})\cdots (x + a_{n})$. Gọi $a_{0}$ là một số thực bất kỳ và đặt $f(x) = (x - a_{0}).g(x) = x^{n + 1} + b_{1}x^{n} + \cdots + b_{n + 1}$. Chứng minh rằng $b_{1}, b_{2}, \cdots b_{n + 1}$ đều âm nếu và chỉ nếu $a_{0} > a_{1} + a_{2} + \cdots a_{n}$




#622491 Việt Nam TST 2016 - Thảo luận đề thi

Đã gửi bởi Ego on 25-03-2016 - 12:17 trong Thi HSG Quốc gia và Quốc tế

To @Mr Stoke, em cảm ơn thầy đã nhận xét cho em. Em cũng thấy lời giải mình nếu dùng Zsigmondy mà lý luận như trên thì đúng là phức tạp thật, hơi nửa nạc nữa mỡ.
Đây là lời giải khác của em

Theo như bổ đề 1 ở trên ta sẽ thu được $\text{gcd}(a^{n} - 1, a^{3^{2016}} - 1) = a^{3^{w}} - 1$ với $w = 2016$ hoặc $w = u$ (trong đó $n = 3^{u}.v$ với $\text{gcd}(3, v) = 1$)

Giả thiết bài toán tương đương nếu $p$ là một số nguyên tố $p\mid a^{n} - 1 \implies p\mid a^{3^{w}} - 1$

  • TH1. $w = u$, khi đó $3^{w}$ là ước của $n$. Nếu $w = 0$ thì áp dụng Zsigmondy thì có một ước nguyên tố $p\mid a^{n} - 1$ mà $p\nmid a^{3^{w}} - 1 = a - 1$ nếu như $v > 2$ hoặc $a \neq 3$. Từ đó ta suy ra $v = 1$ hoặc $v = 2$, riêng với $v = 2$ thì $a = 2^{l} - 1$
    Nếu $w \ge 1$ thì áp dụng Zsigmondy ta lại thu được có một ước nguyên tố $p\mid a^{n} - 1$ mà $p\nmid a^{3^{w}} - 1$ nếu như $v > 1$. Từ đó ta chỉ suy ra $v = 1$.
    Kết luận lại nghiệm, $(a, n) = (3, 2), (t, 3^{u}) \; (u < 2016)$ (điều này đúng do $a^{3^{u}} - 1 \mid a^{3^{2016}} - 1$
  • TH2. $w = 2016$. Tức là $u \ge 2016$. Nếu $v > 1$ hoặc $u > 2016$ thì theo định lý Zsigmondy sẽ có một ước nguyên tố $p\mid a^{n} - 1$ mà $p\nmid a^{3^{2016}} - 1$. Do đó nghiệm là $n = 3^{2016}$.

Vậy nghiệm của bài toán là $(a, n) = (2^{l} - 1, 2), (t, 3^{u})$ với $u\le 2016$.

Spoiler




#622292 Việt Nam TST 2016 - Thảo luận đề thi

Đã gửi bởi Ego on 24-03-2016 - 17:58 trong Thi HSG Quốc gia và Quốc tế

Như Zaraki nói, ta sẽ sử dụng bổ đề $\text{gcd}(a^{m} - 1, a^{n} - 1) = a^{\text{gcd}(m, n)} - 1$
Bổ đề 1. $\text{gcd}(a^{m} - 1, a^{n} - 1) = a^{\text{gcd}(m, n)} - 1$
Chứng minh bổ đề. Gọi $d = \text{gcd}(a^{m} - 1, a^{n} - 1)$
Để ý $a^{\text{gcd}(m, n)} - 1 \mid a^{m} - 1, a^{n} - 1 \implies a^{\text{gcd}(m, n)} - 1 \mid d \implies d \ge a^{\text{gcd}(m, n)} - 1$
Mặt khác $a^{m} \equiv 1 \pmod{d}$ và $a^{n} \equiv 1\pmod{p}$ nên $\text{ord}_{d}(a) \mid m, n \implies \text{ord}_{d}(a) \mid \text{gcd}(m, n)$
Hay $a^{\text{gcd}(m, n)} \equiv 1 \pmod{d}$ hay $a^{\text{gcd}(m, n)} - 1 \ge d$.
Kết hợp lại ta có đpcm.
Bổ đề 2. $3^{3^{u}} + 1 = 2^{t}$ có nghiệm duy nhất là $(u, t) = (0, 2)$.
Chứng minh bổ đề. Xét $u \ge 1$, khi đó $3^{3^{u}} = (3^{3})^{3^{u - 1}} \equiv 3^{3^{u - 1}}\pmod{8}$. Đến đây lùi về sẽ thu được $3^{3^{u}} \equiv 3\pmod{8}$. Từ đó ta có $u = 0$ là nghiệm duy nhất.

Giả thiết bài toán quy thành $p$ là ước nguyên tố của $a^{n} - 1$ thì $p\mid d$ với $d = \text{gcd}(a^{n} - 1, a^{3^{2016}} - 1)$
Ta đặt $n = 3^{u}.v$ với $\text{gcd}(v, 3) = 1$.

  • TH1. $u \le 2016$. Khi đó theo bổ đề $d = a^{3^{u}} - 1$
    i) Nếu $v = 1$ thì ta dễ thấy $a^{3^{u}} - 1 \mid a^{3^{2016}} - 1$.
    ii) Xét $v \ge 2$, khi đó ta có $a^{v} - 1\mid a^{n} - 1$. Bài toán suy ra nếu $p\mid a^{v} - 1$ thì $p\mid a^{3^{u}} - 1$. Xét $p$ là ước nguyên tố của $\frac{a^{v} - 1}{a - 1} \implies p\mid a^{v} - 1$. Khi đó $p\mid \text{gcd}(a^{v} - 1, a^{3^{u}} - 1) = a - 1$ theo bổ đề trên. Điều này có nghĩa là tập các ước nguyên tố của $a^{v - 1} + a^{v - 2} + \cdots + 1$ phải là tập con của tập các ước nguyên tố của $a - 1$ (*)
    Bây giờ ta đặt $A = a^{v - 1} + a^{v - 2} + \cdots + 1 = p_{1}^{d_{1}}p_{2}^{d_{2}}\cdots p_{k}^{d_{k}}$ với $p_{i} < p_{i + 1}$ là các ước nguyên tố.
    Theo (*) thì $a - 1 = p_{1}^{c_{1}}p_{2}^{c_{2}}\cdots p_{k}^{c_{k}}.P$
    a) Nếu $a$ chẵn thì toàn bộ $p_{i}, P$ lẻ. Áp dụng bổ đề LTE, ta có $v_{p_{i}}(A) = v_{p_{i}}(a^{v} - 1) - v_{p_{i}}(a - 1) = v_{p_{i}}(v)$. Nghĩa là $v = p_{1}^{d_{1}}p_{2}^{d_{2}}\cdots p_{k}^{d_{k}}Q$
    Lúc này, $A \ge a^{v - 1} + 1 = a^{p_{1}^{d_{1}}p_{2}^{d_{2}}\cdots p_{k}^{d_{k}}Q - 1} + 1 \ge 3^{p_{1}^{d_{1}}p_{2}^{d_{2}}\cdots p_{k}^{d_{k}}Q - 1} + 1 > A$. Vô lí.
    b) Nếu $a = 4t + 1$. Lúc này ta vẫn có thể áp dụng bổ đề LTE như trên do $4\mid a - 1$ nên $v_{2}(a^{v} - 1) = v_{2}(a - 1) + v_{2}(v)$. Đánh giá tương tự ta có điều vô lý.
    c) Nếu $a = 4t + 3$. Khi đó để ý nếu $v$ lẻ ta sẽ dẫn đến $v_{2}(A) = v_{2}(v) = 0$ nên đánh giá tương tự a) cho điều vô lý.
    Xét $v$ chẵn, theo bổ đề LTE $v_{2}(a^{v} - 1) = v_{2}(a - 1) + v_{2}(a + 1) + v_{2}(v) - 1 = v_{2}(a + 1) + v_{2}(v)$.
    Đặt $L = v_{2}(v)$, $K = v_{2}(A) = d_{1}$ và $J = v_{2}(a + 1) \; (J \ge 2)$. Ta có $K = v_{2}(A) = v_{2}(a^{v} - 1) - v_{2}(a - 1) = L + J - 1$ (lưu ý là do ta đã sắp xếp $p_{i}$ nên $p_{1} = 2$)
    Viết lại $A = 2^{L + J - 1}p_{2}^{d_{2}}\cdots p_{k}^{d_{k}}$ và $v = 2^{L}p_{2}^{d_{2}}\cdots p_{k}^{d_{k}}.Q$. Mặt khác, để ý là $v_{2}(a + 1) = J$ nên $a + 1 \ge 2^{J} \iff a \ge 2^{J} - 1$. Ta có:
    $$A = a^{v - 1} + a^{v - 2} + \cdots + 1 \ge a^{v - 1} + 1 \ge (2^{J} - 1)^{v - 1} + 1 = [(2^{J} - 2) + 1]^{2^{L}p_{2}^{d_{2}}\cdots p_{k}^{d_{k}}Q} + 1 \ge 2^{L}p_{2}^{d_{2}}\cdots p_{k}^{d_{k}}Q(2^{J} - 2) + 2 \ge 2^{L + J}.p_{2}^{d_{2}}\cdots p_{k}^{d_{k}} - 2^{L + 1}p_{2}^{d_{2}}\cdots p_{k}^{d_{k}} + 2 \ge A$$
    Để ý dấu đẳng thức xảy ra khi và chỉ khi $v = 2$, $a = 2^{l} - 1$.
    Từ đó ta cần đi tìm $u$ sao cho mọi ước nguyên tố của $a^{2.3^{u}} - 1$ là ước nguyên tố của $a^{3^{2016}} - 1$. Xét trường hợp $u \neq 0$, theo bổ đề 3 thì $a^{3^{u}} + 1$ có ước nguyên tố lẻ. Xét $q$ là ước nguyên tố lẻ của $a^{3^{u}} + 1$. Ta có $q\mid \frac{a^{3^{2016}} - 1}{a^{3^{u}} - 1}$, nếu ta đặt $x = a^{3^{u}}$ và $y = 3^{2016 - u}$ ($y$ lẻ) thì viết lại $q\mid \frac{x^{y} - 1}{x - 1} = x^{y - 1} + x^{y - 2} + \cdots + 1 = x^{y - 2}(x + 1) + x^{y - 4}(x + 1) + \cdots + x(x + 1) + 1$ (do $y$ lẻ) từ đó suy ra $q\mid 1$ do $q\mid x + 1$. Vô lí.
    Từ đó thu được $u = 0$ hay $n = 2$.
    Thử lại, cho ta bộ nghiệm là $(a, n) = (2^{l} - 1, 2)$
  • TH2. $n > 2016$
    Xét tương tự trên, với $v > 1$ ta cũng sẽ thu được $n = 2$. Vô lí.
    Xét $v = 1$. Khi đó ta cần chứng minh có 1 ước nguyên tố là ước của $3^{3^{u}} - 1$ nhưng không là ước của $3^{3^{2016}} - 1$
    Tuy nhiên điều này không đúng vì nếu ngược lại, nghĩa là sẽ có một ước nguyên tố $q$ sao cho $v_{q}(3^{3^{u}}) > v_{q}(3^{3^{2016}})$. Nếu $q$ lẻ: mọi ước nguyên tố $q$ của $3^{3^{u}} - 1$ và $3^{3^{2016}} - 1$ đều khác $3$ nên theo bổ đề LTE $v_{q}(3^{3^{u}} - 1) = v_{q}(3^{3^{2016}} - 1) + v_{q}(3^{u - 2016}) = v_{q}(3^{3^{2016}})$
    Do đó ta chỉ cần xét $3^{3^{u}} - 1 = 2^{i}.(3^{3^{2016}} - 1)$, tuy nhiên PT này vô nghiệm do đó ta có đpcm (có thể cm bằng cách chia qua) :-)
     

P.S: Hay quá :-s, đoạn cuối đánh giá quên mất việc $a + 1 = 2^{l}$ :-P




#628727 USAMO 2016

Đã gửi bởi Ego on 21-04-2016 - 17:38 trong Thi HSG Quốc gia và Quốc tế

@baopbc: Theo đáp số thì $|S| = 8$. Nhưng để xây dựng được thì có lẽ là hơi khó, em nên xây dựng thử, vì việc em chỉ ra $|S| \ge 8$ mới là điều kiện cần.

@superpower: thành thật xin lỗi bạn và mọi người, mình có sai sót trong khâu đánh lại đề. Hiện tại đề đã được chỉnh sửa.



#628538 USAMO 2016

Đã gửi bởi Ego on 20-04-2016 - 19:12 trong Thi HSG Quốc gia và Quốc tế

USAMO 2016

Ngày 1 (19/04/16)
Bài 1.
Cho $\mathbb{X}_{1}, \mathbb{X}_{2}, \cdots , \mathbb{X}_{100}$ là các tập con khác rỗng đôi một khác nhau của tập $\mathbb{S}$. Hai tập $\mathbb{X}_{i}$ và $\mathbb{X}_{i + 1}$ bất kỳ thì giao của chúng là bằng rỗng và hợp của chúng không là cả một tập $\mathbb{S}$, nói cách khác, $\mathbb{X}_{i} \cap \mathbb{X}_{i + 1} = \varnothing$ và $\mathbb{X}_{i} \cup \mathbb{X}_{i + 1} \neq \mathbb{S}$, với mọi $i \in \{1, \cdots , 99\}$. Tìm số phần tử nhỏ nhất có thể của $\mathbb{S}$
Bài 2. Chứng minh rằng với mọi số nguyên dương $k$, $$(k^{2})!.\prod_{j = 0}^{k - 1}\frac{j!}{(j + k)!}$$ là số nguyên.
Bài 3. Cho $\triangle ABC$ nhọn, và $I_{B}, I_{C}, O$ lần lượt là tâm bàng tiếp $\angle B$, tâm bàng tiếp $\angle C$, tâm ngoại tiếp của $\triangle ABC$. Điểm $E, Y$ được chọn trên $\overline{AC}$ sao cho $\angle ABY = \angle CBY$ và $\overline{BE}\perp \overline{AC}$. Tương tự, Điểm $F, Z$ được chọn trên $\overline{AB}$ sao cho $\angle ACZ = \angle BCZ$ và $\overline{CF}\perp \overline{AB}$. Đường $I_{B}F$ và $I_{C}E$ cắt nhau tại $P$. Chứng minh rằng $\overline{PO} \perp \overline{YZ}$.
Ngày 2 (20/04/16)
Bài 4. Tìm tất cả các hàm $f:\mathbb{R} \to \mathbb{R}$ sao cho với mọi số thực $x, y$, $$(f(x) + xy)f(x - 3y) + (f(y) + xy)f(3x - y) = (f(x + y))^{2}$$
Bài 5. Cho ngũ giác $AMNPQ$ có năm cạnh bằng nhau nội tiếp trong tam giác $ABC$ sao cho $M\in\overline{AB}$, $Q\in\overline{AC}$ và $N, P\in\overline{BC}$. Gọi $S$ là giao điểm của $MN$ và $PQ$. Gọi $(d)$ là đường phân giác của $\angle MSQ$.
Chứng minh rằng $OI \parallel d$ với $O, I$ lần lượt là tâm ngoại, nội tiếp của tam giác $ABC$.
Bài 6. Cho $n, k$ là hai số nguyên, $n \ge k \ge 2$. Bạn tham gia vào một trò chơi với một phù thuỷ ác độc (?!)
Phù thuỷ có $2n$ lá bài, với mỗi $i = 1, 2, \cdots , n$, thì có đúng hai lá bài cùng được đánh số là $i$. Lúc đầu, phù thuỷ sắp úp tất cả lá bài trên một hàng, thứ tự bất kỳ.
Về phần bạn, bạn sẽ lặp lại các thao tác sau: chỉ vào $k$ lá bài bạn muốn chọn, phù thuỷ sẽ lật ngửa lên hộ bạn. Nếu lật lên, có hai tấm thẻ cùng được đánh 1 số, bạn thắng, game sẽ kết thúc. Nếu ngược lại, phù thuỷ sẽ hoán vị $k$ lá bài đó và lật úp chúng lại. Mỗi lần bạn làm vậy là một bước đi. Và bạn sẽ lặp lại thao tác trên.
Ta gọi game này là thành công nếu tồn tại $m$ nguyên dương nào đó và một chiến thuật nào đó ở tối đa $m$ bước đi, không quan trọng việc phù thuỷ xáo trộn bài của bạn.
Hỏi với $n$ và $k$ nào thì game này thành công?

Nguồn



#628531 USA(J)MO 2016

Đã gửi bởi Ego on 20-04-2016 - 18:27 trong Tài liệu - Đề thi

USA(J)MO 2016

Ngày 1. (19/04/2016)
Bài J1. Cho tam giác $ABC$ cân tại $A$ nội tiếp trong đường tròn $\omega$. $P$ là một điểm di động trên cung $BC$ không chứa $A$ của $\omega$ và $I_{b}, I_{c}$ lần lượt là tâm nội tiếp của $\triangle ABP$ và $\triangle ACP$.
Chứng minh rằng khi điểm $P$ di động, đường tròn $(PI_{b}I_{c})$ luôn đi qua một điểm cố định.

Bài J2. Chứng minh rằng tồn tại một số nguyên dương $n < 10^{6}$ sao cho $5^{n}$ chứa sáu chữ số $0$ liên tiếp trong biễu diễn thập phân.

Bài J3. Cho $\mathbb{X}_{1}, \mathbb{X}_{2}, \cdots , \mathbb{X}_{100}$ là các tập con khác rỗng đôi một khác nhau của tập $\mathbb{S}$. Hai tập $\mathbb{X}_{i}$ và $\mathbb{X}_{i + 1}$ bất kỳ thì giao của chúng là bằng rỗng và hợp của chúng không là cả một tập $\mathbb{S}$, nói cách khác, $\mathbb{X}_{i} \cap \mathbb{X}_{i + 1} = \varnothing$ và $\mathbb{X}_{i} \cup \mathbb{X}_{i + 1} \neq \mathbb{S}$, với mọi $i \in \{1, \cdots , 99\}$. Tìm số phần tử nhỏ nhất có thể của $\mathbb{S}$
Ngày 2. (20/04/2016)
Bài 4. Tìm, và chứng minh số nguyên $N$ bé nhất sao cho nếu bỏ đi $2016$ phần tử từ tập $\{1, 2, \cdots , N\}$, chúng ta vẫn có thể tìm ra $2016$ số nguyên phân biệt trong các số còn lại có tổng bằng $N$.
Bài 5. Cho tam giác nhọn $ABC$, tâm ngoại tiếp $O$. $H$ là chân đường cao hạ từ $A$ xuống $BC$ và $P, Q$ lần lượt là chân đường cao hạ từ $H$ xuống $AB, AC$. Cho biết $AH^{2} = 2AO^{2}$. Chứng minh rằng $O, P, Q$ thẳng hàng.
Bài 6. Tìm tất cả các hàm $f:\mathbb{R} \to \mathbb{R}$ sao cho với mọi số thực $x, y$, $$(f(x) + xy)f(x - 3y) + (f(y) + xy)f(3x - y) = (f(x + y))^{2}$$


Nguồn



#628566 USA(J)MO 2016

Đã gửi bởi Ego on 20-04-2016 - 20:59 trong Tài liệu - Đề thi

Bài J2 được đề xuất bởi Evan Chen. Khá hay. Đây là lời giải của mình.
Theo bổ đề LTE, $v_{2}(5^{2^{19}} - 1) = 21$. Do đó $5^{2^{19}} \equiv 1\pmod{2^{21}}$. Đặt $n = 2^{19} + 21$. Từ đó ta có $5^{n} \equiv 5^{21}\pmod{2^{21}}$.
Mặt khác, $5^{n} \equiv 5^{21}\pmod{5^{21}}$ do $n > 20$. Từ đó ta có $5^{n} \equiv 5^{21}\pmod{10^{21}}$.
Gọi $T(n)$ là số chữ số của $n$. Khi đó $5^{n} - 5^{21}$ có $21$ chữ số $0$ tận cùng.
Nhận xét là $14 < \log(5^{14}) < 15$ do đó $T(5^{21}) = 15$, nghĩa là ta có ít nhất $21 - 15 = 6$ số $0$ trong biểu diễn thập phân.

 




#614116 Tặng VMF

Đã gửi bởi Ego on 11-02-2016 - 09:51 trong Đa thức

Bài này mình làm thử vài trường hợp nhỏ rồi tổng quát lên, đưa lên AoPS hỏi cũng chưa có trả lời :-) Hôm nay có ý, đưa mọi người xem thử.
a) Cho tam thức bậc hai $P(x) = ax^{2} + bx + c$ thỏa mãn $|P(x)| \le 1 \; \forall x \in [-1; 1]$. CMR $P'(x) \le 4 \; \forall x \in [-1; 1]$
b) Cho đa thức bậc ba $P(x) = ax^{3} + bx^{2} + c + d$ thỏa mãn $|P(x)| \le 1 \; \forall x \in [-1; 1]$. CMR $P'(x) \le 9 \; \forall x \in [-1; 1]$.
c) Cho đa thức bậc $n$: $P(x) = a_{n}x^{n} + \ldots + a_{0}$ thỏa mãn $|P(x)| \le 1 \; \forall x \in [-1; 1]$. CMR $P'(x) \le n^{2} \; \forall x \in [-1; 1]$




#613993 tìm x,y,z

Đã gửi bởi Ego on 10-02-2016 - 17:51 trong Số học

Trường hợp 1: $z = 1$, có phương trình $(x - y)(x + y) = 2^{100} \implies \begin{cases} x - y = 2^{m} \\ x + y = 2^{n} \end{cases} \implies \begin{cases} x = 2^{m - 1} + 2^{n - 1} \\ y = 2^{n - 1} - 2^{m - 1} \end{cases}$ với $m + n = 100$.
Trường hợp 2: $\begin{cases} z = 5 \\ x = 2 \\ y = 1 \end{cases}$ không thỏa mãn.
Trường hợp 3: Ngoại trừ hai trường hợp trên, nếu $x - y \ge 2$ (lưu ý $x \neq y$) thì áp dụng định lý Zsigmondy cho ta tồn tại một số nguyên tố $p$ là ước của $x^{z + 1} - y^{z + 1}$ nhưng không là ước của $x - y$, vô lí do $x^{z + 1} - y^{z + 1}$ chỉ có ước nguyên tố duy nhất là $2$. Tóm lại, $x - y = 1$. Tuy nhiên lưu ý vế trái $x^{z + 1} - y^{z + 1} \equiv_{2} x - z \equiv_{2} 1$, vô lí do $x^{z + 1} - y^{z + 1}$ chẵn.




#623328 Tìm và chứng minh các giá trị có thể có của đường chéo của bảng vuông

Đã gửi bởi Ego on 28-03-2016 - 23:30 trong Tổ hợp và rời rạc

Viết các số từ $1$ đến $n^{2}$ lên bảng vuông $n\times n$ gồm $n^{2}$ ô vuông đơn vị ($n$ là số tự nhiên cho trước) sao cho mỗi hình chữ nhật tạo từ một số ô vuông đơn vị thì tổng hai số ở hai ô ở hai góc đối diện nhau bằng tổng hai số ở hai ở góc đối diện còn lại. Tìm và chứng minh các giá trị có thể có của đường chéo của bảng vuông.

Nguồn: SL Challenge Competition

Spoiler




#621506 Tìm tất cả số nguyên dương $n$ sao cho $p\mid \dbino...

Đã gửi bởi Ego on 20-03-2016 - 21:21 trong Số học

Cho $p$ là một số nguyên tố. Tìm tất cả số nguyên dương $n$ sao cho $p\mid \dbinom{n}{k}$ với mọi $k = 1, 2, \cdots, n - 1$.

Spoiler




#629839 Tìm tất cả hàm liên tục $f:\mathbb{R}\to\mathbb...

Đã gửi bởi Ego on 27-04-2016 - 18:29 trong Phương trình hàm

Tìm tất cả hàm liên tục $f:\mathbb{R}\to\mathbb{R}$ sao cho $$f(f(x)) = f(x) + x$$ với mọi số thực $x$

Nguồn




#630505 TURKEY Team Selection Test 2016

Đã gửi bởi Ego on 01-05-2016 - 11:36 trong Thi HSG Quốc gia và Quốc tế

TURKEY Team Selection Test 2016

Ngày 1 (02/04/2016)
Bài 1. Cho tam giác $ABC$ nhọn, $P$ thuộc đường cao hạ từ $A$. Các đường thẳng $BP, CP$ lần lượt cắt các cạnh $CA, AB$ tương ứng tại $D, E$. Tiếp tuyến kẻ từ $D$ và $E$ tới $(BPC)$ tiếp xúc tại $K, L$ tương ứng ($K, L$ nằm trong tam giác). Đường thẳng $KD$ giao với $(AKC)$ tại $M$ ($M \neq K$) và đường $LE$ giao với $(ALB)$ tại $N$ ($N \neq L$). Chứng minh rằng $\frac{DK}{DM} = \frac{EL}{EN}$ khi và chỉ khi $P$ là trực tâm của tam giác $ABC$.

Bài 2. Trong một lớp học có $23$ học sinh, mỗi cặp học sinh đã coi một bộ phim cùng nhau. Với mỗi học sinh, các bộ phim học sinh đó coi gọi là tuyển tập các bộ phim của học sinh đó.  Biết rằng mỗi học sinh đã coi mỗi bộ phim tối đa một lần, hỏi có ít nhất bao nhiêu tuyển tập các bộ phim khác nhau của những học sinh này?

Bài 3. Cho $a, b, c$ là các số thực không âm thỏa mãn $a^{2} + b^{2} + c^{2} \le 3$. Chứng minh rằng $$(a + b + c)(a + b + c - abc) \ge 2(a^{2}b + b^{2}c + c^{2}a)$$

Ngày 2 (03/04/2016)
Bài 4. Một dãy các số thực $a_{0}, a_{1}, \cdots$ thỏa mãn điều kiện $$\sum_{n = 0}^{m}a_{n}.(-1)^{n}.\binom{m}{n} = 0$$ với mọi số nguyên dương $m$ đủ lớn. Chứng minh rằng tồn tại một đa thức $P$ thỏa mãn $a_{n} = P_{n}$ với mọi $n \ge 0$.

Bài 5. Tìm tất cả hàm $f:\mathbb{N}\to\mathbb{N}$ sao cho với mọi $m, n \in \mathbb{N}$ thì $f(mn) = f(m).f(n)$ và $m + n \mid f(m) + f(n)$

Bài 6. Cho tam giác $ABC$ cân tại $A$ với $D$ là trung điểm của $BC$. Một đường thẳng qua $D$ cắt $AB$ tại $K$, $AC$ tại $L$. Điểm $E$ trên cạnh $BC$ ($E \neq D$), $P$ trên $AE$ sao cho $\angle KPL = 90^{\circ} - \frac{1}{2}\angle KAL$; $E$ nằm giữa $A$ và $P$. Đường tròn ngoại tiếp tam giác $PDE$ cắt $PK$ tại $X$ ($X \neq P$) và cắt $PL$ tại $Y$ ($Y \neq L$). Đường $DX$ cắt $AB$ tại $M$, đường $DY$ cắt $AC$ tại $N$. Chứng minh rằng bốn điểm $P, M, A, N$ cùng thuộc một đường tròn.

Ngày 3 (04/04/2016)
Bài 7. Cho $A_{1}, A_{2}, \cdots , A_{k}$ là các tập con khác nhau của tập $\{1, 2, \cdots , 2016\}$. Nếu $A_{i} \cap A_{j}$ tạo thành một cấp số cộng với mỗi $1\le i \le j \le 2016$. Tìm giá trị lớn nhất có thể của $k$.

Bài 8. Các góc của một đa giác lồi $A_{1}A_{2}\cdots A_{n}$ ($n \ge 5$) là góc tù. Với mọi $1 \le i \le n$, $O_{i}$ là tâm ngoại tiếp của $A_{i - 1}A_{i}A_{i + 1}$ (quy ước $A_{0} = A_{n}$ và $A_{1} = A_{n + 1}$). Chứng minh rằng $O_{1}O_{2}\cdots O_{n}$ không phải là một $n$ - giác lồi.

Bài 9. $p$ là một số nguyên tố. Đặt $\mathbb{K}_{p}$ là tập hợp tất cả các đa thức hệ số nguyên thuộc tập $\{0, 1, \cdots , p - 1\}$ và bậc nhỏ hơn $p$. Giả sử với mọi cặp đa thức $P, Q \in \mathbb{K}_{p}$ sao cho $P(Q(n)) \equiv n\pmod{p}$ với mọi số nguyên $n$, thì bậc của $P$ và $Q$ là bằng nhau. Xác định mọi số nguyên tố $p$ thỏa mãn điều kiện trên.

Nguồn




#630590 TURKEY Team Selection Test 2016

Đã gửi bởi Ego on 01-05-2016 - 19:41 trong Thi HSG Quốc gia và Quốc tế

Mà $c^2\leq 1\Rightarrow b^2\geq 1$

Tại sao bạn có điều này, $b \le 1$ vẫn có lí chứ, khi đó $bc \le 1$ và $b^{2} + c^{2} \le 2$ vẫn đúng mà :)
Và bất đẳng thức $a + b + c - abc - 2 \ge 0$ không đúng đâu ạ. Ví dụ $a = b = c = \frac{1}{3}$ thì sai rõ ràng luôn.




#646114 TRƯỜNG HÈ TOÁN HỌC MIỀN BẮC 2016

Đã gửi bởi Ego on 23-07-2016 - 12:29 trong Thi HSG cấp Tỉnh, Thành phố. Olympic 30-4. Đề thi và kiểm tra đội tuyển các cấp.

Bài 1. Cho 2016 tập hợp, mỗi tập có 45 phần tử, hai tập bất kì có đúng một phần tử chung. Chứng minh rằng tồn tại 1 phần tử thuộc tất cả các tập.




#634783 Topic về tổ hợp, các bài toán về tổ hợp

Đã gửi bởi Ego on 22-05-2016 - 20:44 trong Tổ hợp và rời rạc

Mở hàng.
Bài 50. (Đồ thị) Cho một graph $G$ có $n$ đỉnh, sao cho không có đỉnh nào có bậc lớn hơn $k$. Chứng minh rằng ta có thể tô màu các đỉnh với tối đa $k + 1$ màu, sao cho hai đỉnh kề nhau thì chung màu.




#615384 Tìm tất cả các số nguyên dương n sao cho (n-1)! chia hết cho $...

Đã gửi bởi Ego on 16-02-2016 - 17:33 trong Số học

Mấy nay bị nhắc nhở không được post bài :)) Buồn thật..
Quay lại bài toán,

i) Ta sẽ chứng minh là nếu $n$ có nhiều hơn $3$ ước nguyên tố thì $(n - 1)! \vdots n^{2}$. Thật vậy, xét $n = \prod_{i = 1}^{k}p_{i}^{a_{i}}$ với $k \ge 3$, $p_{1} < p_{2} < \ldots < p_{k}$ và $a_{i}$ là các số nguyên dương (cái này cm bằng quy nạp được)
Để ý là $p_{g}^{a_{g}}.p_{h}^{a_{h}} \ge p_{1}^{a_{1}}.p_{2}^{a_{2}} \ge 6$ và $3.2^{\alpha} > 3\alpha$ với mọi $\alpha$ là số nguyên dương.
Xét số nguyên tố $p_{i}$ bất kì, khi đó theo định lý Legendre
$$v_{p_{i}}[(n - 1)!] = v_{p_{i}}(n!) - v_{p_{i}}(n) = v_{p_{i}}(n!) - a_{i} \ge 6.p_{i}^{a_{i} - 1} - a_{i} \ge 3.2^{a_{i}} - a_{i} \ge 2a_{i}$$
Vậy từ đó có điều phải chứng minh.
ii) Xét $n = p^{a}.q^{b}$ với $a, b$ là hai số nguyên dương và $p < q$ là hai số nguyên tố.
Nếu $p \ge 3$ thì trong các số từ $1$ đến $n - 1$ gồm có số $(p^{a} - 1).q^{b}$ và $(p^{a} - 2).q^{b}$. Do đó $v_{q}[(n - 1)!] \ge 2b$. Chứng minh tương tự cũng có được $v_{p}[(n - 1)!] \ge 2a$. (cái này chứng minh bằng Legendre được nhưng dài dòng quá mình lí luận vậy cho lẹ)
Nếu $p = 2$ và $a \ge 2$ thì cũng tương tự trên (do $2^{2} > 2$)
Vậy ta chỉ xét $n = 2.q^{b}$ (Nhận xét là $2.3^{b - 1} \ge 3b$ với mọi $b$ nguyên dương không bé hơn $2$), nếu $q \ge 2$ thì $$v_{q}[(n - 1)!] = v_{q}(n!) - v_{q}(n) = v_{q}(n!) - b \ge [\frac{n}{q}] - b \ge 2.q^{b - 1} - b \ge 2.3^{q - 1} - b \ge 3b - b = 2b$$. Dễ thấy $v_{2} > 2$ nên $(n - 1)! \vdots n^{2}$. Vậy ta chỉ xét $n = 2.q$ với $q \ge 3$. Ta sẽ chứng minh số này không thỏa mãn.
$v_{q}[(n - 1)!] = v_{q}(n!) - v_{q}(n) = 2 - 1 = 1$.
iii) Xét $n = p^{a}$. Rõ ràng nếu $a = 1$ thì không thỏa. Xét $a \ge 2$
- Nếu $p = 2$ thử từ i), ii) cho ta $a \ge 4$. Ta sẽ chứng minh $a \ge 4$ thì thỏa mãn: (để ý $2^{a} - 1 \ge 3a$ với $a \ge 4$)
Khi đó theo Legendre thì $v_{2}[(n - 1)!] = 1 + 2 + \ldots + 2^{a - 1} - a = 2^{a} - 1 \ge 2a$. Đúng.
- Nếu $p = 3$. Thử với $9$, ta thấy không thỏa mãn. Xét $a \ge 3$, (có $3^{a} - 1 \ge 6a$ với mọi $a \ge 3$)
$v_{3}[(n - 1)!] = v_{3}(n!) - a = 1 + 3 + \ldots + 3^{a - 1} - a = \frac{3^{a} - 1}{2} - a \ge 2a$, xong.
Vậy các số thỏa mãn đề bài là các số không thuộc tập hợp $(8, 9, p, 2p)$ với $p$ là một số nguyên tố bất kỳ.




#616630 Tìm n để n! là số chính phương

Đã gửi bởi Ego on 23-02-2016 - 22:34 trong Số học

Nếu được phép áp dụng bổ đề luôn thì đây là lời giải của mình :-D
Ta sẽ chứng minh chỉ có $n = 1$ là thỏa mãn. Kiểm tra được $n = 1$ là nghiệm. Giả sử tồn tại $n \ge 2$ sao cho $n! = m^{2}$.
Bổ đề. Cho $p$ là số nguyên tố sao cho $p\mid K^{2}$ thì $p^{2} \mid K^{2}$
Gọi $q$ là số nguyên tố lớn nhất không vượt quá $n$. Khi đó do $q\mid m^{2}$ áp dụng bổ đề ta có $q^{2}\mid n!$. Nói cách khác, trong các số từ 1 đến $n$ có chứa số $2q$.
Mặt khác, áp dụng bổ đề Bertrand, giữa $q$ và $2q$ có một số nguyên tố $p$ sao cho $q < p < 2q$. Điều này mâu thuẫn với tính lớn nhất của $q$.